Calculating Max Output Power from a 12V Battery w/ 6 Ohm Resistance

Click For Summary
The maximum output power from a 12V battery with a 6-ohm resistance is calculated using the formula P = VI, resulting in 24 watts based on a current of 2 amperes. However, the internal resistance of the battery must be considered, which leads to a maximum power output of 6 watts according to some sources. Additionally, when analyzing a sine wave with a peak-to-peak voltage of 7 volts, the average value is calculated to be approximately 2.23 volts. If averaged over the entire cycle, the average value would be zero. The discussion highlights the importance of accounting for internal resistance and the correct formulas in power calculations.
Manelectro
Messages
3
Reaction score
0
I'm having a tough time wrapping my head around something that seems so simple. The Question is:

What is the maximum output power from a 12 volt battery that has an internal resistance of 6 ohm ?

The answer from this books is 6 W?

My formula
First I need to find the current
I = V/R so 12volt/6 ohm = 2 ampere

When is use this formula
P = VI, 12Vx2A = 24Watt,

The next question is:
On an oscilloscope display of a pure sine wave , the peak to peak value is 7 volts. The average value of the display voltage is about:

A. 6.36 volt
B.3.18 volt
C. 0 volt
D. 2.23 volt

The answer from this book is 0 volt.

The peak to peak voltage is 7/2 = 3.5V this peak voltage

But by calculation Vaverage = 2 x 3.5/ pi = 2.228 or 2.23 V for sine wave or AC wave formula.

If the average value were taken over the whole cycle then the result would be zero value.

Thank you,
Azman
 
Physics news on Phys.org
Anybody can answer this question. where the math guru...anyway thank :)
 
delet
 
Question: A clock's minute hand has length 4 and its hour hand has length 3. What is the distance between the tips at the moment when it is increasing most rapidly?(Putnam Exam Question) Answer: Making assumption that both the hands moves at constant angular velocities, the answer is ## \sqrt{7} .## But don't you think this assumption is somewhat doubtful and wrong?

Similar threads

  • · Replies 10 ·
Replies
10
Views
4K
Replies
1
Views
2K
  • · Replies 7 ·
Replies
7
Views
2K
  • · Replies 5 ·
Replies
5
Views
2K
  • · Replies 1 ·
Replies
1
Views
2K
  • · Replies 17 ·
Replies
17
Views
5K
  • · Replies 15 ·
Replies
15
Views
11K
  • · Replies 16 ·
Replies
16
Views
3K
  • · Replies 17 ·
Replies
17
Views
4K
  • · Replies 5 ·
Replies
5
Views
4K